DOC PREVIEW
TAMU MATH 409 - oct6

This preview shows page 1 out of 4 pages.

Save
View full document
View full document
Premium Document
Do you want full access? Go Premium and unlock all 4 pages.
Access to all documents
Download any document
Ad free experience
Premium Document
Do you want full access? Go Premium and unlock all 4 pages.
Access to all documents
Download any document
Ad free experience

Unformatted text preview:

Math 409-502Harold P. [email protected] of convergence tests• If an6→ 0, then∑nandiverges.• Comparison tests for positive series: if an≤ bnfor all large n, or alternatively if an/bnhasa finite limit, then convergence of∑nbnimplies convergence of∑nan.• Absolute convergence implies convergence: if∑n|an| converges, then so does∑nan.• Ratio and root tests: if either limn→ ∞|an|1/nor limn→ ∞|an+1/an| exists and is strictly less than 1,then∑nanconverges.• Special tests for decreasing positive terms an:(i) if f (x) ↓ 0 as x → ∞, thenR∞f (x) dx and∑∞f (n) have the same conver-gence/divergence behavior (integral test); (ii) if an↓ 0, then∑n(−1)nanconverges (al-ternating series test).Math 409-502 October 6, 2004 — slide #2Cauchy’s condensation testAnother special test for decreasing termsSuppose 0 < an+1≤ anfor all (large) n. Then the two series∑nanand∑n2na2neither bothconverge or both diverge.Example:∑n1n ln(n)Since1n ln(n)is a decreasing function of n, the test applies and says that the conver-gence/divergence behavior is the same as for the series∑n2n12nln(2n). That simplifies to∑n1n ln(2), which is a multiple of the divergent harmonic series. Therefore the original series∑n1n ln(n)diverges too.Math 409-502 October 6, 2004 — slide #3Proof of the condensation test (sketch)a8+ a9+ ···+ a15≤ 8a8≤ 2(a4+ a5+ a6+ a7)a16+ a17+ ···+ a31≤ 16a16≤ 2(a8+ a9+ ···+ a15)...Adding such inequalities shows that partial sums of∑n2na2nare bounded below by partialsums of∑nanand are bounded above by twice the partial sums of∑nan. Therefore the twoseries have the same convergence/divergence behavior.Math 409-502 October 6, 2004 — slide #4Power seriesExample:∞∑n=1xn2n√nFor which values of x does that series converge?[This is Exercise 8.1/1a on page 123.]Solution:By the root test, the series converges (absolutely) when1 > limn→ ∞¯¯¯¯xn2n√n¯¯¯¯1/n= limn→ ∞|x|2√n1/n=|x|2,that is, when |x| < 2.The series diverges when |x| > 2 by the (proof of the) root test.A different test is needed to see what happens when x = ±2.Math 409-502 October 6, 2004 — slide #5Homework• Read section 8.1, pages 114–117.• Do Exercise 7.6/1a,c on page 111.• Do Exercise 8.1/1g on page 123.Math 409-502 October 6, 2004 — slide


View Full Document

TAMU MATH 409 - oct6

Documents in this Course
nov17

nov17

4 pages

sep17

sep17

3 pages

nov22

nov22

4 pages

nov10

nov10

5 pages

oct13

oct13

4 pages

nov8

nov8

3 pages

oct29

oct29

4 pages

nov15

nov15

4 pages

oct22

oct22

4 pages

nov12

nov12

3 pages

oct11

oct11

4 pages

dec03

dec03

4 pages

sep20

sep20

4 pages

final

final

2 pages

oct18

oct18

4 pages

nov5

nov5

3 pages

Load more
Download oct6
Our administrator received your request to download this document. We will send you the file to your email shortly.
Loading Unlocking...
Login

Join to view oct6 and access 3M+ class-specific study document.

or
We will never post anything without your permission.
Don't have an account?
Sign Up

Join to view oct6 2 2 and access 3M+ class-specific study document.

or

By creating an account you agree to our Privacy Policy and Terms Of Use

Already a member?